You are on page 1of 82

1.

Consider the following differences between

Parasitism and Predation:

1. A parasite kills a prey whereas a

predator does not cause death to the

host.

2. In parasitism the weak feed on the strong

while in predation the strong feed on the

weak.

Which of the statements given above is/are

correct?

(a) 1 only

(b) 2 only

(c) Both 1 and 2

(d) Neither 1 nor 2

Q 1.B

• Predation and parasitism are two types of negative biotic interactions between the organisms
living in the

Earth.

o Parasitism is an interaction between a parasite and a host in which the former obtains benefits at
the

cost of later. A parasite does not cause the death of the host. Whereas Predation is an association

between two species, where one species (predator) kills to feed on the prey. Hence, statement 1 is

not correct.

o In Parasitism the weak feed on the strong while in predation it is strong that feeds on the weak.

Hence, statement 2 is correct.

o For example, Glochidium larva attaches to the fins of a fish is an example of parasitism and Birds

feeding on fish is an example of predation.


2. With reference to the productivity of an

ecosystem, consider the following

statements:

1. Gross primary productivity of an

ecosystem is the rate of production of

organic and inorganic matter.

2. Net primary productivity is the available

biomass for consumption to

heterotrophs.

Which of the statements given above is/are

correct?

(a) 1 only

(b) 2 only

(c) Both 1 and 2

(d) Neither 1 nor 2

Q 2.B

• The Gross primary productivity of an ecosystem is the rate of production of organic matter during

photosynthesis. Hence, statement 1 is not correct.

• A considerable amount of GPP is utilized by plants in respiration. Gross primary productivity minus

respiration losses (R), is the net primary productivity (NPP).

o GPP – R = NPP

• Net primary productivity is the available biomass for the consumption of heterotrophs (herbivores
and

decomposers). Hence, statement 2 is correct.

• Secondary productivity is defined as the rate of formation of new organic matter by consumers.

• Primary productivity depends on the plant species inhabiting a particular area. It also depends on a
variety

of environmental factors, availability of nutrients and photosynthetic capacity of plants. Therefore, it


varies in different types of ecosystems.

• The annual net primary productivity of the whole biosphere is approximately 170 billion tons (dry

weight) of organic matter. Of this, despite occupying about 70 percent of the surface, the
productivity of

the oceans is only 55 billion tons. The remaining productivity is on land.

3. Which of the following are effective

methods in dealing with air pollution?

1. Washing of coal

2. Replacement of controlled oxygen

furnaces with open-hearth furnaces

3. Use of storage tanks with floating roof

covers in petroleum refineries

Select the correct answer using the code

given below.

(a) 1 and 2 only

(b) 1 and 3 only

(c) 2 and 3 only

(d) 1, 2 and 3

Q 3.B

• Option 1 is correct. Change of process is one of the source correction methods used in the
abatement of

air pollution. Changing or modifying the process of production can help in lowering atmospheric

pollutants. For example, coal is washed before pulverizing it to reduce fly ash emissions.

• Option 2 is not correct and option 3 is correct. Existing equipment can be modified to
reduce/control

undesirable emissions.

o If open-hearth furnaces are replaced with controlled basic oxygen/electric furnaces then the

emission of carbon monoxide, smoke and fumes can be reduced.


o In petroleum refineries, loss of hydrocarbon vapours from storage tanks (due to evaporation or

temperature change) can be reduced by designing storage tanks with floating roof covers.

4. In the context of ecology, Antibiosis refers

to

(a) competition between organisms of the

same species.

(b) an association between two species,

where one species kills to feed on the

other.

(c) a relationship in which one organism is

benefited by direct utilization of another.

(d) secretions by an organism which are

harmful to others.

Q 4.D

• Organisms living in this earth are interlinked to each other in one way or other. The interaction
between

the organisms is fundamental for the survival and functioning of the ecosystem as a whole.

• Antibiosis is a negative interaction in which an organism produces harmful secretions. In this type

of relationship, none of the population is benefited. Hence, option (d) is correct.

• For example, some species of blue-green algae that grows in ponds produce toxic substances that
kilt

fishes as well. In marine waters, the population of some microbes popularly known as red tide cause
the

destruction of fish and other animals.

• Intraspecific competition is the competition between organisms of the same species.

• Predation is an association between two species, where one species kills to feed on the other.

• Exploitation is the relationship in which one organism is benefited by direct utilization of another
5. Which of the following statements is correct

regarding the air pollution control device,

scrubber?

(a) It separates particulate matter from gases

in electric power plants.

(b) It allows settling of the pollutants in a

gas to settle down due to the effect of

gravity.

(c) It employs spray of water to catch

pollutants during emissions.

(d) It imparts electrical charge to the

particles that makes them stick to the

metallic plates.

Q 5.C

• A Filter separates particulate matter from gases in electric power plants. The smoke passes
through a

series of cloth bags which trap the particulate matter.

• Gravitational Settling Chamber consists of huge rectangular chambers. The polluted gas stream is

allowed to enter from one side. The horizontal velocity of the gas stream is kept at a very low level
to give

time for the particles to settle down by gravity.

• The scrubber is a device that uses a spray of water to catch pollutants during emissions. A dry
scrubber is

used to remove acid gases. This process includes spraying wet lime powder into the hot exhaust
chamber.

The scrubber uses this lime to neutralise acid gases. Hence option (c) is the correct answer.

• Electrostatic Precipitator helps in controlling pollution by removing many chemicals. It removes the

particles from the smoke only after the combustion has taken place. It imparts an electrical charge to
the
particles that make them stick to the metallic plates inside the precipitator.

6. Which of the following is/are the features of

Grassland Ecosystems?

1. Grasslands have a rich variety of

animals.

2. Optimum conditions for the growth of

trees

3. Soil in grasslands are characterized by

thin podzols.

Select the correct answer using the code

given below.

(a) 1 only

(b) 2 and 3 only

(c) 1 and 3 only

(d) 2 only

Q 6.A

• Grasslands are found on every continent except Antarctica. These terrestrial ecosystems occupy
roughly

19 percent of the earth's surface. They are characterised by treeless herbaceous plants dominated
by a

wide variety of grass

• Features of Grasslands:

o The grasslands are found where rainfall is about 25-75 cm per year, not enough to support a

forest, but more than that of a true desert. Typical grasslands are vegetation formations that are

generally found in temperate climates.

o Grasslands are dominated by grasses. Large shrubs or trees are not found. Hence, statement 2 is

not correct.
o Grasslands have a rich variety of animals. Hence, statement 1 is correct.

o The soil is always exposed, sometimes rocky but more often sandy with fixed or mobile dunes.

Forage is available only during the brief wet season. Boreal forest soils are characterized by thin

podzols. Hence, statement 3 is not correct.

o Grasslands have been the home for grazing animals for millions of years.

7. Which of the following are the components

of Photochemical Smog?

1. Ozone

2. Formaldehyde

3. Peroxyacetyl Nitrate (PAN)

Select the correct answer using the code

given below.

(a) 1 and 2 only

(b) 2 and 3 only

(c) 1 and 3 only

(d) 1, 2 and 3

Q 7.D

• Photochemical smog is a mixture of pollutants that are formed when nitrogen oxides and volatile

organic compounds (VOCs) react to sunlight, creating a brown haze above cities. It occurs in warm,
dry,

and sunny climates. The main components of the photochemical smog result from the action of
sunlight

on unsaturated hydrocarbons and nitrogen oxides produced by automobiles and factories.

Photochemical smog has a high concentration of oxidizing agents and is, therefore, called oxidizing

smog.

• Formation of Photochemical Smog:

• When fossil fuels are burnt, a variety of pollutants are emitted into the earth’s troposphere
including
hydrocarbons (unburnt fuels) and nitric oxide (NO). When these pollutants build up to sufficiently
high levels, a chain reaction occurs from their interaction with sunlight in which NO is converted into
nitrogen

dioxide (NO2).

o This NO2 in turn absorbs energy from sunlight and breaks it up into nitric oxide and free oxygen

atoms.

✓ NO2(g) → NO(g) + O(g)

o Oxygen atoms are very reactive and combine with the O2 in the air to produce ozone.

✓ O(g) + O2 (g) → O3 (g)

o The ozone formed in the above reaction reacts rapidly with the NO to regenerate NO2. NO2 is a
brown

gas and at sufficiently high levels can contribute to haze.

✓ NO(g)+O (g)→NO (g)+O (g)

• Ozone is a toxic gas and both NO2 and O3 are strong oxidizing agents and can react with the
unburnt

hydrocarbons in the polluted air to produce chemicals such as formaldehyde, acrolein, and
peroxyacetyl

nitrate (PAN).

• Effects of Photochemical Smog:

• The common components of photochemical smog are ozone, nitric oxide, acrolein, formaldehyde,
and

peroxyacetyl nitrate (PAN). Photochemical smog causes serious health problems. Both ozone and
PAN

act as powerful eye irritants. Ozone and nitric oxide irritate the nose and throat and their high

concentration causes headache, chest pain, dryness of the throat, cough and difficulty in breathing.
Hence

option (d) is the correct answer.

8. Consider the following statements with

respect to ozone depletion:

1. Chlorofluorocarbons (CFCs) are nontoxic organic molecules that help in


ozone depletion.

2. Polar stratospheric clouds provide a

surface for the reactions causing ozone

depletion.

3. Free Chlorine radicals form when CFCs

are broken down by UV radiations.

Which of the statements given above are

correct?

(a) 1 and 2 only

(b) 2 and 3 only

(c) 1 and 3 only

(d) 1, 2 and 3

Q 8.D

• Ozone in the stratosphere is a product of UV radiations acting on dioxygen (O2) molecules. The UV

radiations split apart molecular oxygen into free oxygen (O) atoms. Ozone is thermodynamically

unstable and decomposes to molecular oxygen. Thus, a dynamic equilibrium exists between the

production and decomposition of ozone molecules.

• In recent years, there have been reports of the depletion of this protective ozone layer because of
the

presence of certain chemicals in the stratosphere. The main reason for ozone layer depletion is the
release

of chlorofluorocarbon compounds (CFCs), also known as freons. These compounds are nonreactive,

nonflammable, nontoxic organic molecules and therefore used in refrigerators, air conditioners, in
the

production of plastic foam, and by the electronic industry for cleaning computer parts, etc. Once
CFCs are

released into the atmosphere, they mix with the normal atmospheric gases and eventually reach the

stratosphere. In the stratosphere, they get broken down by powerful UV radiations, releasing

chlorine-free radicals. The chlorine radicals then react with stratospheric ozone to form chlorine
monoxide radicals and molecular oxygen. The reaction of chlorine monoxide radical with atomic
oxygen

produces more chlorine radicals. The chlorine radicals are continuously regenerated and cause the

breakdown of ozone. Thus, CFCs are transporting agents for continuously generating chlorine

radicals into the stratosphere and damaging the ozone layer. Hence statements 1 and 3 are correct.

• In winter, a special type of cloud called polar stratospheric cloud is formed over Antarctica. These
polar

stratospheric clouds provide a surface on which chlorine nitrate formed gets hydrolyzed to form

hypochlorous acid. It also reacts with hydrogen chloride produced as per reaction (v) to give
molecular

chlorine. When sunlight returns to Antarctica in the spring, the sun’s warmth breaks up the clouds,

and HOCl and Cl2 are photolyzed by sunlight. The chlorine radicals thus formed, initiate the chain

reaction for ozone depletion as described earlier. Hence statement 2 is correct.

9. Which of the following are the sources of

Nitrogen in an ecosystem?

1. Agricultural land

2. Fossil fuel

3. Livestock

4. Urban waste

Select the correct answer using the code

given below.

(a) 1 and 4 only

(b) 2, 3 and 4 only

(c) 1, 2 and 3 only

(d) 1, 2, 3 and 4

Q 9.D

• Burning fossil fuels, application of nitrogen-based fertilizers, and other activities can dramatically
increase the amount of biologically available nitrogen in an ecosystem. The main sources of nitrogen
in

the atmosphere therefore include:

o Atmospheric precipitation,

o Geological sources,

o Agricultural land,

o Livestock (ruminants like cows, sheep, goats etc) and poultry operations and

o Urban waste

• Hence option (d) is the correct answer.

10. India’s single largest solar park by NTPC

(National Thermal Power Corporation) is set

to come up in the state of

(a) Gujarat

(b) Rajasthan

(c) Tamil Nadu

(d) Uttar Pradesh

Q 10.A

• Recently, NTPC Renewable Energy Ltd, a 100% subsidiary of NTPC, has received a go-ahead

under 'Ultra Mega Renewable Energy Power Park of the Solar Park Scheme' from the Ministry of

New and Renewable Energy to set up a 4750 MW renewable energy park at Rann of Kutch in

Khavada, Gujarat. This will be India’s largest solar park to be built by the largest power producer

of the country.

• About Ultra Mega Renewable Energy Power Park of the Solar Park Scheme

o The scheme for “Development of Solar Parks and Ultra Mega Solar Power Projects” was rolled out
by

the Ministry of New & Renewable Energy on 12-12-2014.

o Under this scheme, it was proposed to set up at least 25 Solar Parks and Ultra Mega Solar Power
Projects targeting over 20,000 MW of solar power installed capacity within a span of 5 years starting

from 2014-15.

o The capacity of the Scheme has been enhanced from 20,000 MW to 40,000 MW in 2017.

• NTPC REL also plans to generate green hydrogen on a commercial scale from this park.

• As a part of its green energy portfolio augmentation, NTPC Ltd aims to build 60 GW Renewable
Energy

Capacity by 2032. Currently, the state-owned power major has an installed capacity of 66 GW across
70

power projects with an additional 18 GW under construction.

• Recently, NTPC has also commissioned India’s largest Floating Solar of 10 MW (ac) on the reservoir
of

Simhadri Thermal Power Plant, Andhra Pradesh.

• Hence option (a) is the correct answer.

11. Which of the following is/are greenhouse

gases (GHG)?

1. Water vapor

2. Carbon Dioxide

3. Nitrous Oxide

Select the correct answer using the code

given below.

(a) 1 and 2 only

(b) 2 only

(c) 1 and 3 only

(d) 1, 2 and 3

Q 11.D

• The greenhouse effect is a natural process that warms the Earth’s surface. When the Sun’s energy

reaches the Earth’s atmosphere, some of it is reflected back to space and the rest is absorbed and
reradiated by greenhouse gases. Earth is surrounded by a blanket of air called the atmosphere.
Greenhouse
gas molecules in the atmosphere trap heat as they are transparent to sunlight but not to heat
radiation. If

the amount of carbon dioxide crosses the delicate proportion of 0.03 percent, the natural
greenhouse

balance may get disturbed. Carbon dioxide is a major contributor to global warming.

• Besides carbon dioxide, other greenhouse gases are methane, water vapor, nitrous oxide, and

ozone. Methane is produced naturally when vegetation is burnt, digested, or rotted in the absence
of oxygen. Large amounts of methane are released in paddy fields, coal mines, rotting garbage
dumps, and

by fossil fuels. Chlorofluorocarbons (CFCs) are man-made industrial chemicals used in air
conditioning

etc. CFCs are also damaging the ozone layer. Nitrous oxide occurs naturally in the environment. In
recent

years, their quantities have increased significantly due to the use of chemical fertilizers and the
burning of

fossil fuels.

• Hence option (d) is the correct answer.

12. Consider the following statements with

respect to oxides of carbon:

1. Carbon Monoxide is produced as a result

of the incomplete combustion of carbon.

2. Carbon dioxide forms approximately 3%

by volume of the atmosphere.

Which of the statements given above is/are

correct?

(a) 1 only

(b) 2 only

(c) Both 1 and 2

(d) Neither 1 nor 2


Q 12.A

• Carbon oxides, or oxocarbons, are a class of organic compounds containing only carbon and
oxygen. The

most basic oxocarbons are carbon monoxide and carbon dioxide.

• Carbon monoxide: Carbon monoxide (CO) is a colorless and odorless gas, highly poisonous to living

beings because of its ability to block the delivery of oxygen to organs and tissues. It is produced as

a result of the incomplete combustion of carbon. CO is poisonous as it binds to hemoglobin to

form carboxyhemoglobin, which is about 300 times more stable than the oxygen-hemoglobin

complex. In blood, when the concentration of carboxyhemoglobin reaches about 3–4 percent, the
oxygencarrying capacity of blood is greatly reduced. This oxygen deficiency results in headaches,
weak eyesight,

nervousness, and cardiovascular disorder. In pregnant women who have the habit of smoking the

increased CO level in blood may induce premature birth, spontaneous abortions, and deformed

babies. Hence statement 1 is correct.

• Carbon dioxide: Carbon dioxide (CO2) is released into the atmosphere by respiration, burning of
fossil

fuels for energy, and by decomposition of limestone during the manufacture of cement. It is also
emitted

during volcanic eruptions. Normally it forms about 0.03 percent by volume of the atmosphere.
Hence

statement 2 is not correct.

• With the increased use of fossil fuels, a large amount of carbon dioxide gets released into the
atmosphere.

Green plants require CO2 for photosynthesis and they, in turn, emit oxygen, thus maintaining the
delicate

balance. Deforestation and the burning of fossil fuels increase the CO2 level and disturb the balance
in the

atmosphere. The increased amount of CO2 in the air is mainly responsible for global warming.

13. Consider the following statements with

respect to ozone:
1. Tropospheric ozone is a greenhouse gas.

2. Ozone in the stratosphere is a product of

UV radiation acting on dioxygen (O2)

molecules.

3. Ozone is thermodynamically stable than

oxygen.

Which of the statements given above are

correct?

(a) 1 and 3 only

(b) 2 and 3 only

(c) 1 and 2 only

(d) 1, 2 and 3

Q 13.C

• Ground level or tropospheric ozone is created by chemical reactions between oxides of nitrogen
(NOx

gases) and volatile organic compounds (VOCs). Ozone in the troposphere is considered a greenhouse

gas and contributes to global warming. It is also a common constituent of Photochemical smog.
Hence

statement 1 is correct.

• The upper stratosphere consists of a considerable amount of ozone (O3), which protects us from
the

harmful ultraviolet (UV) radiations coming from the sun. These radiations cause skin cancer
(melanoma)

in humans. Therefore, it is important to maintain the ozone shield. Ozone in the stratosphere is a
product

of UV radiations acting on dioxygen (O2) molecules. The UV radiations split apart molecular oxygen

into free oxygen (O) atoms. These oxygen atoms combine with molecular oxygen to form ozone.
Hence

statement 2 is correct.
• Ozone is thermodynamically less stable than oxygen and decomposes into molecular resulting in
the

liberation of heat. Ozone consists of three molecules of oxygen and is thus in an unstable state. So in
order

to get stable, it gives up one molecule of oxygen to restore the diatomic state. Thus, a dynamic

equilibrium exists between the production and decomposition of ozone molecules. In recent years,
there

have been reports of the depletion of this protective ozone layer because of the presence of certain

chemicals in the stratosphere. The main reason for ozone layer depletion is believed to be the
release

of chlorofluorocarbon compounds (CFCs), also known as freons. Hence statement 3 is not correct.

14. Consider the following statements regarding

Biogas:

1. It is produced by the fermentation of

biodegradable materials such as

biomass, manure, municipal waste and

green waste in the presence of air.

2. It primarily consists of methane and

carbon dioxide.

3. It has very low calorific value and

cannot be used as a fuel

Which of the statements given above is/are

not correct?

(a) 1 only

(b) 2 and 3 only

(c) 1 and 3 only

(d) 2 only

Q 14.C
• Biogas refers to a gas produced by the biological breakdown of organic matter in the absence of

oxygen. Hence statement 1 is not correct.

• Biogas is produced by the anaerobic digestion or fermentation of biodegradable materials such as

biomass, manure or sewage, municipal waste, green waste and energy crops. This type of biogas

comprises methane and carbon dioxiode. The other type is wood gas which is created by gasification
of

wood or other biomass.

• Biogas is primarily composed of methane gas, carbon dioxide, and trace amounts of nitrogen,
hydrogen,

and carbon monoxide. Hence statement 2 is correct. Gobar gas is produced from the anaerobic
digestion

of manure.

• Importance of Biogas:

o The products of biogas plant, methane gas is used as fuel and liquid humus as natural fertilizer.

o Biogas can be generated from locally available materials like animal dung, agricultural waste etc.

o It is a clean fuel.

o It has high calorific value. Hence statement 3 is not correct.

15. Consider the following pairs:

Type of Pollutant Disease Caused

1. Arsenic : Cholera

2. Mercury : Itai-Itai

3. Cadmium : Minamata Disease

Which of the pairs given above is/are

correctly matched?

(a) 1 only

(b) 1 and 2 only

(c) 2 and 3 only

(d) None
Q 15.D

• Pair 1 is not correctly matched. Wastewater including sewage contains many disease-causing

pathogens. Polluted water (and not Arsenic) can cause many diseases like cholera, typhoid, diarrhea,

dysentery, polio, and jaundice. Cholera is an acute diarrheal illness caused by infection of the

intestine with Vibrio Cholerae bacteria

• Pair 2 is not correctly matched. Pollution by heavy metal cadmium caused the Itai- Itai disease in

Japan.

• Pair 3 is not correctly matched. Mercury dumped into water is transformed into water-soluble

methyl mercury by bacterial action. This methyl mercury accumulates in the fish. Minamata

disease was caused on a large scale due to the consumption of this methyl mercury-contaminated
fish.

16. Consider the following:

1. Styrofoam CD and DVD cases

2. Leather bags

3. Plastic toys

4. Synthetic pesticides

Which of the above mentioned waste

products fall in the category of nonbiodegradable wastes?

(a) 1, 2 and 4 only

(b) 1, 3 and 4 only

(c) 2 and 3 only

(d) 1, 2, 3 and 4

Q 16.B

• Wastes can be classified into various categories like biodegradable and non - biodegradable
wastes.

• Wastes that can be degraded or broken down through microbial activities of fungi and bacteria are
called biodegradable wastes. Agricultural wastes, faecal remains, dead plants, leather shoes/bags,
tin

cans falls under this category.

• Wastes which cannot be degraded or broken down through microbial activities are called
nonbiodegradable wastes. Such wastes include crude petroleum, plastics, styrofoam products,

glasses, polymer, synthetic pesticides, radioactive fall out etc.

o Styrofoam is a pure solid, hard with limited flexibility. It is used for making disposable cutlery,
plastic

models, CD and DVD cases etc.

• Hence option (b) is the correct answer.

17. India has been ranked 10th in the latest

Global Cybersecurity Index (GCI), 2020.

GCI is released by

(a) Global Cyber Alliance

(b) International Telecommunication Union

(c) Internet Society

(d) International Cyber Security Protection

Alliance

Q 17.B

• The Global Cybersecurity Index (GCI) is a multi-stakeholder initiative to raise cybersecurity


awareness

and to measure the commitment of countries to cybersecurity and its wide field of application
cutting

across industries and sectors.

o It is released by the International Telecommunication Union which is a specialized agency of the

United Nations responsible for all matters related to information and communication

technologies.

• India has been ranked 10th in Global Cybersecurity Index (GCI) 2020. In the Asia Pacific region
India has been ranked 4th.

• USA topped the list by securing first position followed by UK and Saudi Arabia together.

• Global Cybersecurity Index ranking is based on five pillars :

o Legal measures

o Technical measures

o Capacity building measures

o Organizational measures

o Cooperation

• Hence, option (b) is the correct answer.

18. "It is a Greenhouse gas. It is released in large

amounts in paddy fields, coal mines, from

rotting garbage dumps. It is associated with

chemical reactions leading to the destruction

of ozone. It is also used as biogas." It is

(a) Nitrous Oxide

(b) Methane

(c) Chlorofluorocarbon

(d) Carbon dioxide

Q 18.C

• The Convention on the Conservation of Migratory Species of Wild Animals (Bonn Convention; CMS)
is

an environmental treaty under the aegis of the United Nations Environment Programme. Hence,

statement 2 is correct.

• It provides a global platform for the conservation and sustainable use of migratory animals and
their

habitats. CMS is the only global and UN-based intergovernmental organisation established

exclusively for the conservation and management of terrestrial, aquatic and avian migratory species
throughout their range. Hence, statement 1 is correct.

• Migratory species threatened with extinction are listed on Appendix I of the Convention. CMS
Parties

strive towards strictly protecting these animals, conserving or restoring the places where they live,

mitigating obstacles to migration and controlling other factors that might endanger them. Besides

establishing obligations for each State joining the Convention, CMS promotes concerted action
among the

Range States of many of these species.

• Migratory species that need or would significantly benefit from international co-operation are
listed in

Appendix II of the Convention. For this reason, the Convention encourages the Range States to
conclude

global or regional agreements.

19. Consider the following statements about the

National Board for Wildlife:

1. It is a statutory body, established under

the Wildlife Protection Act, 1972.

2. It is headed by the Minister-in-charge of

Environment, Forest, and Climate

Change (MoEFCC).

Which of the statements given above is/are

correct?

(a) 1 only

(b) 2 only

(c) Both 1 and 2

(d) Neither 1 nor 2

Q 19.A
• The National Board for Wildlife (NBWL) is a statutory body, established under the Wildlife
Protection

Act, 1972. It is important to point out that the wildlife act, as originally enacted in 1972, did not
provide

for the NBWL. It was only through an amendment of the Wildlife Act in 2002 that the NBWL was

constituted. It is a 47-member committee. Hence statement 1 is correct.

o The Prime Minister as Chairperson. Hence statement 2 is not correct.

o The Minister in charge of Environment, Forest and Climate Change (MoEFCC) as ViceChairperson.

o The standing committee of NBWL is chaired by the Minister of Environment Forest and Climate

Change.

• Functions of the National Board:

o It is the apex advisory body which looks at the implementation of various schemes for wildlife

conservation and development of wildlife and forests by such measures as it thinks fit. It is not

entrusted with dealing with matters of environmental pollution.

o Framing policies and advising the Central Government and the State Governments on the ways
and

means of promoting wildlife conservation and effectively controlling poaching and illegal trade of

wildlife and its products.

o Making recommendations on the setting up of and management of national parks, sanctuaries,


and

other protected areas and on matters relating to the restriction of activities in those areas.

o Carrying out or causing to be carried but impact assessment of various projects and activities on

wildlife or its habitat.

o Review from time to time, the progress in the field of wildlife conservation in the country and

suggesting measures for improvement thereto.

o Preparing and publishing a status report at least once in two years on wildlife in the country.

20. Which of the following organisms act as a

biological agent in increasing the plant


growth of farmland?

1. Praying Mantis

2. Desert Locusts

3. Chalcid Wasps

4. Soldier Beetle

Select the correct answer using the code

given below.

(a) 1, 2, 3 and 4

(b) 1, 3 and 4 only

(c) 1 and 4 only

(d) 2 and 4 only

Q 20.B

• Desert locusts are known for causing massive destruction of food crops, greenery, and plants. They

reproduce in lakhs and form swarms in search of food. They travel hundreds of miles with their
strong

wings and legs without taking any break. According to FAO, a one square kilometer swarm of locusts,

with about 40 million locusts, can in a day eat as much food as 35,000 people, assuming that each

individual consumes 2.3 kg of food per day. Hence only option 2 is not correct

• Praying mantises prey and eat any living organism they can successfully capture and devour. But
insects

may be herbivores, neutrals and carnivores form the main diet. They eat beetles, weevils, bugs,
moths,

butterflies, and various insects available in our crops. Thus they act as natural pesticides for

controlling the various leaf-eating loopers and semi-loopers, fruit-eating bollworms, leaf folders, and

stem borers, bugs including notorious mealybug, weevils like a grey weevil, and beetles like chefer,

brown flower beetle, and insect pests like jassids, aphids, hoppers.

• Chalcid Wasps are insects within the superfamily Chalcidoidea, part of the order Hymenoptera.
The
superfamily contains some 22,500 known species, and an estimated total diversity of more than
500,000

species, meaning the vast majority have yet to be discovered and described. Generally beneficial to

humans as a group, chalcidoids help keep various crop pests under control, and many species have

been imported as biocontrol agents.

• Soldier beetles are a common outdoor insect that can be abundant in accidental invaders as either
larvae

or adults. Soldier beetles are nicknamed leatherwings because of their soft, clothlike wing covers,
which

when brightly colored are reminiscent of uniforms. The soldier beetle life cycle begins as a larva that

hatches from an egg in the fall. These larvas are predators and will eat the eggs of many garden
pests,

as well as damaging larvae and soft insect bodies. They then hibernate in the soil or among fallen

leaves until spring.

• Hence option (b) is the correct answer

21. These are called the "rainforests of the

ocean". They cannot survive above water

level and are mostly confined between 25

degrees North and South on the globe. They

are generally attached to submarine

platforms of islands that are submerged.

Which of the following is best described by

the passage given above?

(a) Phytoplanktons

(b) Mangroves

(c) Coral Reefs

(d) Seagrass

Q 21.C
• Coral reefs are often called the "rainforests of the sea" for their astounding richness of life. Due to
their

structural complexity, corals are one of the most productive ecosystems on Earth, providing
important

services to mankind including fisheries, coastal protection, medicines, recreation, and tourism.

• Corals are tiny animals that live in colonies and derive nourishment and energy from a symbiotic

relationship with zooxanthellae algae. Coral reefs are formed over the course of thousands of years
as

limestone skeletons constructed by corals accumulate and form a structural base for living corals.

Scientists estimate reefs provide a home for millions of species - from brightly colored tropical fish to
sea

cucumbers which produce anti-cancer compounds. The formation of highly consolidated reefs only
occurs

where the temperature does not fall below 18°C for extended periods of time. The water must also
be

clear to permit high light penetration. The corals’ requirement for high light also explains why most
reefbuilding species are restricted to the euphotic (light penetration) zone, approximately 70 m. In
light of

such stringent environmental restrictions, reefs generally are confined to tropical and semitropical
waters.

• The diversity of reef corals, i.e., the number of species, decreases in higher latitudes up to about
25° north

and south, beyond which reef corals are usually not found. Bermuda, at 32° north latitude, is an
exception

to this rule because it lies directly in the path of the Gulf Stream’s warming waters.

• Hence option (c) is the correct answer

22. Which of the following measures can be

taken for the abatement of soil pollution?

1. Sanitary Landfills

2. Increased use of pesticides

3. Composting
Select the correct answer using the code

given below.

(a) 1 and 2 only

(b) 2 only

(c) 1 and 3 only

(d) None of the above

Q 22.C

• Option 1 is correct. A Sanitary landfill is a way of disposing of refuse on land without creating a

nuisance to public health. Here the waste is dumped in a site and covered with earth to prevent
rodents or

insects from entering into it. The waste is then subjected to bacterial decomposition.

• Option 2 is not correct. An increase in the use of pesticides is harmful to the soil and leads to soil

pollution. Instead, biological methods of pest control can be used to decrease the need for
pesticides.

• Option 3 is correct. Composting waste is an aerobic method of decomposing solid wastes. It


involves

the decomposition of waste into humus called compost which acts as a good fertilizer for plants. The

microorganisms help to stabilize the organic matter. Example Fungi starts working in the first week
after

dumping the material. Actinomycetes help in the last stages of the breakdown. Bacteria is present all

throughout the process. It involves the decomposition of waste into humus called compost which
acts as a

good fertilizer for plants.

23. With reference to the ecosystem, which of

the following correctly describes the

'standing state'?

(a) Amount of dry biomass present in an

ecosystem at any given time


(b) The total number of microorganism

species in an ecosystem.

(c) Amount of inorganic nutrients found in

an ecosystem.

(d) Amount of annual rainfall received

within the boundaries of an ecosystem

Q 23.C

• Standing State: It is the amount of inorganic nutrients found in an ecosystem such as carbon,
nitrogen,

phosphorus, calcium, etc. Hence, option (c) is the correct answer.

• It represents the part of non-living matter.

• It varies in different kinds of ecosystems and also on a seasonal basis.

• It usually occurs in the growth medium of producers. It determines the productivity of the
ecosystem.

• It circulates between living and non-living components of the ecosystem.

• It is being regularly depleted and replenished by the living matters.

• Standing crop is the amount of biomass present in an ecosystem. It represents the entire living
matter.

24. Which of the following is/are the adaptation

features found in plants and animals of the

Desert ecosystem?

1. Hard and thick waxy leaves.

2. Long and spreading root systems

3. Concentrated body fat

4. Overactive sweat glands

Select the correct answer using the code

given below.

(a) 2 and 4 only


(b) 1 and 3 only

(c) 1, 2 and 4 only

(d) 1, 2 and 3 only

Q 24.D

• Desert plants have had to develop different ways of capturing water in order to survive in their
habitat.

These changes are called adaptation.

• A common adaptation to store water in the roots, stems, leaves or fruit. Plants that store water in
this way

are called succulents, one of which is the cactus.

• Some plants have developed very long roots that go deep into the ground to reach underground
water.

Others have developed spreading root systems lying just below the surface and stretching widely.
This

gives the plant many tiny roots that capture water when it rains. Hence option 2 is correct.

• Another desert adaptation is seen in the leaves. Desert plants limit water loss through their surface
leaves

by the size or texture of their leaves.

• Small or spiny leaves limit the surface area exposed to the drying heat. Glossy leaves reflect the
sun's

rays, reducing leaf temperatures. Waxy leaves prevent moisture from escaping. Hence option 1 is

correct.
• In order to survive, desert animals have developed a number of ways of adapting to their habitat.
The most

common adaptation in behaviour is staying in the shade of plants or rocks or by burrowing


underground in

the heat of the day. Many desert animals are nocturnal i.e., they stay inactive in shelter during the
day and

hunt at night when it is cool.

• Fat increases body heat, so some desert animals have concentrated the body's fat in one place,
such as a

hump or tail, rather than having it all through the body. Hence option 3 is correct.
• The absence of sweat glands and the concentration of urine are other physical adaptations made
by

desert animals. Hence, statement 4 is not correct.

25. Which of the following can be determined

by counting the pugmarks in a defined area?

1. Presence of different species in the area.

2. Age of different species.

3. Sex ratio of different species

4. Identification of Individual animals

Select the correct answer using the code

given below.

(a) 1, 2 and 4 only

(b) 3 and 4 only

(c) 1 and 2 only

(d) 1, 2, 3 and 4

Q 25.D

• Pugmark is the term used to refer to the footprint of most animals. These are the marks which are
left by

different animal species while they are walking, running, or moving from one place to another

place. Pugmark of every individual animal species is distinct it is used for identification purposes.
They

are used foro Tracking of animals (especially large cats).

o To make an accurate identification of the presence of different species

o accurate determination of sex

o Accurate identification of age and physical condition of an animal.

• Hence option (d) is the correct answer.

26. With reference to Genetic Biodiversity,


consider the following statements:

1. It refers to the variation of genes among

different species.

2. Higher genetic biodiversity leads to

lesser flexibility in adaptation to

environmental changes.

Which of the statements given above is/are

correct?

(a) 1 only

(b) 2 only

(c) Both 1 and 2

(d) Neither 1 nor 2

Q 26.D

• Genetic diversity refers to the diversity (or genetic variability) within a single species. Each

individual species possesses genes that are the source of its own unique features: In human beings,
for

example, the huge variety of people's faces reflects each person's genetic individuality. The term
genetic

diversity also covers distinct populations of a single species, such as the thousands of breeds of
different

dogs or the numerous variety of roses. Hence statement 1 is not correct.

• Human beings genetically belong to the homo sapiens group and also differ in their characteristics
such as

height, colour, physical appearance, etc., considerably. This is due to genetic diversity

• The huge variety of different gene sets define an individual or a whole population's ability to
tolerate

stress from any given environmental factor. Genetic Diversity is also important with respect to the

adaptability of species to varied environments with special reference to changing climatic conditions.
So
higher is the Genetic diversity greater are the chances of adapting to the Environmental changes and
thus

more confirmed is the survival of the species. Hence statement 2 is not correct.

27. Consider the following statements with

reference to the flow of energy in a food

chain:

1. Energy flow in a food chain is always

unidirectional in nature

2. The loss of energy at each trophic level

is in the range of 10-15 percent.

3. Higher the trophic level the greater is the

available energy.

Which of the statements given above is/are

correct?

(a) 1 and 2 only

(b) 2 and 3 only

(c) 1 only

(d) 1, 2 and 3

Q 27.C

• Organisms in the ecosystem are related to each other through feeding mechanisms or trophic
levels, i.e.

one organism becomes food for the other. A sequence of organisms that feed on one another, form
a food

chain.

• Energy is the basic force responsible for all metabolic activities. The flow of energy from producer
to

top consumers is called energy flow which is unidirectional. Hence, statement 1 is correct.

• The transfer of food energy from its source in plants through a series of organisms with repeated
eating
and being eaten is referred to as a food chain. At each transfer a lot of energy (80-90%) from food is

lost as heat. Hence, only the remaining 10-20% is transferred to the next trophic level. Hence,

statement 2 is not correct.

• Thus, each trophic level contains less energy than the preceding level. For example, deer
(herbivores) are

more abundant than tigers {carnivores). Therefore, the number of steps in any food chain is limited
to four

or five. The nearer the organism is to the beginning of the food chain, the greater is the available

energy. And higher the trophic level, the lesser the energy available. Hence, statement 3 is not

correct.

28. Which of the following characteristics are

associated with a eutrophic lake?

1. Low amount of nutrients

2. Loss of Biodiversity

3. High amount of dissolved oxygen

4. Excessive growth of algae

Select the correct answer using the code

given below.

(a) 2 and 4 only

(b) 1 and 3 only

(c) 1, 2 and 4 only

(d) 2, 3 and 4 only

Q 28.A

• Eutrophication is the process in which a water body becomes overly enriched with nutrients,
leading to

the plentiful growth of simple plant life. The excessive growth (or bloom) of algae and plankton in a

water body are indicators of this process. Eutrophication is considered to be a serious environmental

concern since it often results in the deterioration of water quality and the depletion of dissolved
oxygen in water bodies. Eutrophic waters can eventually become “dead zones” that are incapable of

supporting life.

• Chemical pollutants in water and Eutrophication process:

o As water is an excellent solvent, water-soluble inorganic chemicals that include heavy metals such
as

cadmium, mercury, nickel, etc constitute an important class of pollutants. Organic chemicals are

another group of substances that are found in polluted water. Petroleum products pollute many

sources of water e.g., major oil spills in oceans. Other organic substances with serious impacts are
the

pesticides that drift down from sprays or runoff from lands. Various industrial chemicals

like polychlorinated biphenyls, (PCBs) which are used as cleaning solvents, detergents, and

fertilizers add to the list of water pollutants. PCBs are suspected to be carcinogenic.

o Nowadays most of the detergents available are biodegradable. However, their use can create
other

problems. The bacteria are responsible for degrading biodegradable detergent feed on it and grow

rapidly. While growing, they may use up all the oxygen dissolved in water. The lack of oxygen kills

all other forms of aquatic life such as fish and plants. Fertilizers contain phosphates as additives. The

addition of phosphates in water enhances algae growth. Such profuse growth of algae covers the

water surface and reduces the oxygen concentration in water. This leads to anaerobic conditions,

commonly with the accumulation of obnoxious decay and animal death. Thus, bloom-infested water

inhibits the growth of other living organisms in the water body. This process in which
nutrientenriched water bodies support a dense plant population, which kills animal life by depriving
it of

oxygen and results in subsequent loss of biodiversity is known as Eutrophication.

o Hence option (a) is the correct answer

29. The Animal Welfare Board was established

under which of the following Act?

(a) The Wildlife Protection Act, 1972


(b) The Biological Diversity Act, 2002

(c) The Prevention of Cruelty to Animals

Act, 1960

(d) The Environment Protection Act, 1986

Q 29.C

• The Animal Welfare Board of India is a statutory advisory body on Animal Welfare Laws and
promotes

animal welfare in the country. The Animal Welfare Board of India, headquartered at Ballabhgarh in

Haryana state, is a statutory advisory body advising the Government of India's Ministry of Fisheries,

Animal Husbandry, and Dairying. It was previously based in Chennai.

• It was established in 1962 under Section 4 of the Prevention of Cruelty to Animals Act, 1960.

• It was started under the stewardship of Late Smt. Rukmini Devi Arundale, well known
humanitarian.

From ensuring that animal welfare laws in the country are diligently followed, to provide grants to
Animal Welfare Organizations and advising the Government of India on animal welfare issues, the
Board has

been the face of the animal welfare movement in the country for the last 50 years.

• The Board consists of 28 Members. The term of office of Members is for a period of 3 years.

• The Prevention of Cruelty to Animals Act, 1960:

o It is to prevent the infliction of unnecessary pain or suffering on animals.

o It provides for punishment for causing unnecessary cruelty and suffering to animals.

o It defines animals and different forms of animals.

o It discusses different forms of cruelty, exceptions, and the killing of a suffering animal in case any

cruelty has been committed against it, so as to relieve it from further suffering.

o It provides the guidelines relating to experimentation on animals for scientific purposes.

o It enshrines the provisions relating to the exhibition of the performing animals, and offenses

committed against the performing animals.

o It provides for the limitation period of 3 months beyond which no prosecution shall lie for any

offenses under this Act.


• Hence option (c) is the correct answer.

30. Mahananda river which is an important

tributary of river Ganga flows through the

states of

1. Bihar

2. Uttar Pradesh

3. West Bengal

Select the correct answer using the code

given below.

(a) 1 only

(b) 2 only

(c) 1 and 3 only

(d) 1, 2 and 3

Q 30.C

• Mahananda Bachao Committee has recently filed a case at the National Green Tribunal with

regard to the Mahananda river pollution.

o Rapid urbanization and a lack of space to dispose of waste have led the Mahananda river to be
treated

as a dumpsite.

o Rampant encroachment along the river bed and illegal sale of land has further infringed the quality
of

the river.

• Mahananda River is a transboundary river that flows through the Indian states of Bihar and West

Bengal. It is an important tributary of the river Ganga.

• It is spread over districts of Purnea, Kishanganj, Araria and Katihar in Bihar and Malda, West

Dinajpur and Darjeeling in West Bengal.

• The Mahananda originates from the Paglajhora falls near Kurseong in West Bengal’s Darjeeling
district.
The water — clean and pristine during origin — turns filthy and polluted as it enters Siliguri in

Champasari.

• Mahananda River merges into the Ganga at Godagari Ghat in Bangladesh after flowing for 360

kilometres.

• The main tributaries of the Mahananda river are Balason, Mechi, Kankai.

• Hence, option (c) is the correct answer.

31. Which of the following causes are

responsible for Desertification?

1. Lack of vegetation cover

2. Water erosion

3. Slash and burn agriculture

4. Overgrazing

Select the correct answer using the code

given below.

(a) 1, 2 and 3 only

(b) 2 and 4 only

(c) 1, 3 and 4 only

(d) 1, 2, 3 and 4

Q 31.D

• Desertification is the reduction or destruction of the biological potential of the land, leading to
desert like

conditions.

• Lack of vegetative cover, wind and water erosion, overgrazing, slash and burn agriculture cause

desertification.

• Meaningful agricultural activities are not possible on desert land. Desertification causes land
degradation,

making the soil infertile.


• Hence option (d) is the correct answer.

32. Consider the following statements with

reference to the Indian Forest Act, 1927:

1. It regulates the movement of forest

produce, and the duty leviable on forest

produce.

2. It explains the procedure to be followed

for declaring an area as Reserved Forest.

3. The State Government can assign to any

village community the rights of

government over any land of a reserve

forest.

Which of the statements given above is/are

correct?

(a) 3 only

(b) 1 and 3 only

(c) 1 and 2 only

(d) 1, 2 and 3

Q 32.D

• The purpose of the Indian Forest Act,1927 was the protection and conservation of forests and
judicial use

of forest products. It aimed to regulate the movement of forest produce, and duty leviable forest

produce. Hence statement 1 is correct.

o It also explains the procedure to be followed for declaring an area as Reserved Forest, Protected

Forest or a Village Forest. Hence statement 2 is correct.

✓ Reserved Forests: Reserve forests are the most restricted forests and are constituted by the State

Government on any forest land or wasteland which is the property of the Government. In reserved
forests, local people are prohibited, unless specifically allowed by a Forest Officer in the course

of the settlement.

✓ Protected Forests: The State Government is empowered to constitute any land other than

reserved forests as protected forests over which the Government has proprietary rights and the

power to issue rules regarding the use of such forests. This power has been used to establish State

control over trees, whose timber, fruit or other non-wood products have revenue-raising potential.
✓ Village forest: Village forests are the one in which the State Government may assign to any

village community the rights of Government to or over any land which has been constituted a

reserved forest. Hence statement 3 is correct.

o The State Government may declare a tree or trees in a protected forest as reserved.

o The State Government may prohibit the quarrying of stone or the burning of lime or charcoal or

collection or removal of any forest produce in any forest.

33. Which of the following pollutants can be

classified as toxic particulate material?

1. Lead

2. Cadmium

3. Mercury

4. Asbestos

5. Nickel

Select the correct answer using the code

given below.

(a) 1, 3 and 5 only

(b) 1, 2, 3 and 4 only

(c) 1, 4 and 5 only

(d) 1, 2, 3, 4 and 5

Q 33.D
• Toxic particulate materials are those material which contaminate food and water supply posing a
severe

threat to public health.

o Lead is a harmful toxin and it can affect the development of a child's brain.

o Asbestos can cause a respiratory disease known as asbestosis, as well as chest and lung cancer.

o Mercury is a highly toxic chemical which attacks the nervous system, causing brain damage and
ever

death.

o Nickel can cause respiratory symptoms and lung cancer.

o Cadmium can cause cardiovascular diseases and hypertension.

o Arsenic is another toxic chemical that has been shown to cause cancer.

• Hence option (d) is the correct answer.

34. Which of the following statements is not

correct with respect to acid rain?

(a) Acid rain is resulted from the emission

of oxides of sulphur and nitrogen.

(b) Acid rain dissolves and washes away

nutrients needed for the growth of

crops.

(c) Acid rain has a pH value of between 6

and 7.

(d) Acid rain corrodes water pipes resulting

in the leaching of heavy metals into

drinking water.

Q 34.C

• Acid rain, or acid deposition, is a broad term that includes any form of precipitation with acidic

components, such as sulfuric or nitric acid that fall to the ground from the atmosphere in wet or dry
forms.
Normally rainwater has a pH of 5.6 due to the presence of H+ ions formed by the reaction of
rainwater

with carbon dioxide present in the atmosphere. When the pH of the rainwater drops below 5.6, it is

called acid rain. Hence option (c) is the correct answer.

• Acid rain refers to the ways in which acid from the atmosphere is deposited on the earth’s surface.
Oxides

of nitrogen and sulfur which are acidic in nature can be blown by the wind along with solid particles
in

the atmosphere and finally settle down either on the ground as dry deposition or in water, fog and
snow as

wet deposition.

• Acid rain is a byproduct of a variety of human activities that emit the oxides of sulfur and nitrogen
in the

atmosphere. As mentioned earlier, the burning of fossil fuels (which contain sulfur and nitrogenous

matter) such as coal and oil in power stations and furnaces or petrol and diesel in motor engines
produce

sulfur dioxide and nitrogen oxides. SO2 and NO2 after oxidation and reaction with water are major

contributors to acid rain because polluted air usually contains particulate matter that catalyzes the

oxidation.

• Ammonium salts are also formed and can be seen as an atmospheric haze (aerosol of fine
particles).

Aerosol particles of oxides or ammonium salts in raindrops result in wet deposition. SO2 is also
absorbed

directly on both solid and liquid ground surfaces and is thus deposited as dry deposition.

• Acid rain is harmful to agriculture, trees, and plants as it dissolves and washes away nutrients
needed

for their growth. It causes respiratory ailments in human beings and animals. When acid rain falls
and

flows as groundwater reaches rivers, lakes, etc. it affects plants and animal life in the aquatic
ecosystem.

It corrodes water pipes resulting in the leaching of heavy metals such as iron, lead, and copper into
the
drinking water.

35. Which of the following states in India have

potential for wind energy generation?

1. Andhra Pradesh

2. Gujarat

3. Maharashtra

4. Tamil Nadu

Select the correct answer using the code

given below.

(a) 1, 2 and 3 only

(b) 2 and 3 only

(c) 3 and 4 only

(d) 1, 2, 3 and 4

Q 35.D

• India has huge potential for wind energy generation. Large areas having annual average wind
speeds in

excess of 20 km per hour are available in Tamil Nadu, Andhra Pradesh, Kerala and Karnataka and

the coastal states of Gujarat and Maharashtra.

• Wind energy has also been utilized by Indian technologists through the creation of small water
pumping

wind mills called Samira, which can be used as effective rural technology to lift water from shallow
wells,

irrigation through pumps, heating of water etc.

• Hence option (d) is the correct answer.

36. Which of the following statements is/are

correct about biodiversity hotspots?

1. It represents floral and faunal endemicity


in a region.

2. It is declared by the World Wide Fund

for Nature (WWF) in association with

the International Union for Conservation

of Nature (IUCN).

Select the correct answer using the code

given below.

(a) 1 only

(b) 2 only

(c) Both 1 and 2

(d) Neither 1 nor 2

Q 36.A

• Norman Myers in 1988 has put forth the concept of Biodiversity Hotspot. A region is declared as a

biodiversity hotspot region when two strict criteria are met: Species endemism - it must contain at
least

1,500 species of vascular plants as endemics which is to say that it must have a high percentage of
plant

life found nowhere else on the planet.

• Degree of threat - it has to have lost at least 70% of its original habitat which means, it must be

threatened.
• Statement 1 is correct: Each biodiversity hot spot represents a remarkable universe of
extraordinary

endemicity of both plants and animals species struggling to survive in rapidly shrinking ecosystems.
More

than 50% of the world’s plant species and 42% of all terrestrial animal species are endemic to the 36

biodiversity hot spots.

• Statement 2 is not correct: Conservation International has been working for over 30 years to
protect

biodiversity hotspots through cutting-edge science, innovative policy, and global reach. It also
empowers
people to protect the nature that the world relies on for food, freshwater, and livelihoods.

37. Consider the following statements about the

effect of frost on the growth of plants:

1. It increases the concentration of salts in

the cell.

2. It leads to the formation of canker

disease.

Which of the statements given above is/are

correct?

(a) 1 only

(b) 2 only

(c) Both 1 and 2

(d) Neither 1 nor 2

Q 37.C

• Abiotic or non-living and biotic or living are two components of the ecosystem. They are the same
for

both the ecosystem and the environment. These factors determine the survival of an organism and
one

single factor can limit the range of an organism.

• Abiotic factors include- Energy, Radiation, Temperature & heat flow, Water, Frost Atmospheric
gases

and wind, Fire, Gravity, Topography, Soil, Geologic substratum, etc.

• Impact of Frost on plant growth:

o It results in freezing the soil moisture.

o There is an increasing concentration of salts and dehydration of cells because Water in the

intercellular spaces of the plant gets frozen into ice. Hence statement 1 is correct.

o It leads to the formation of canker disease. (Bacteria, fungi, and viruses are the reason behind
other diseases having similar symptoms). Hence statement 2 is correct.

38. The National Green Tribunal (NGT) has the

jurisdiction to settle disputes related to

which of the following acts?

1. Wildlife Protection Act, 1972

2. Biological Diversity Act, 2002

3. Public Liability Insurance Act, 1991

4. Indian Forest Act, 1927

Select the correct answer using the code

given below.

(a) 1, 2 and 3 only

(b) 1 and 4 only

(c) 2 and 3 only

(d) 1, 2, 3 and 4

Q 38.C

• The National Green Tribunal Act, 2010 was enacted to create a special tribunal, the National Green

Tribunal, to handle the expeditious disposal of the cases pertaining to environmental issues.

• It draws inspiration from the constitutional provision of Article 21 for the Protection of life and
personal

liberty, which assures the citizens of India the right to a healthy environment.

• The Tribunal's dedicated jurisdiction in environmental matters provide speedy environmental


justice and

help reduce the burden of litigation in the higher courts.

• It is not be bound by the procedure laid down under the Code of Civil Procedure, 1908, but shall be

guided by principles of natural justice.

• The Tribunal shall have the jurisdiction over all civil cases where a substantial question relating to

environment (including enforcement of any legal right relating to environment), is involved and such
question arises out of the implementation of the enactments specified in Schedule I as follows:

o The Water (Prevention and Control of Pollution) Act, 1974;

o The Water (Prevention and Control of Pollution) Cess Act, 1977;

o The Forest (Conservation) Act, 1980;

o The Air (Prevention and Control of Pollution) Act, 1981;

o The Environment (Protection) Act, 1986;

o The Public Liability Insurance Act, 1991;

o The Biological Diversity Act, 2002.

• Hence, option (c) is the correct answer.

39. Whenever forest land is diverted for nonforest purposes, Compensatory Afforestation

is mandatory under which of the following

Acts?

(a) Biological Diversity Act, 2002

(b) Forest (Conservation) Act, 1980

(c) Environment (Protection) Act, 1986

(d) Indian Forest Act, 1927

Q 39.B

• In India, Forest land can be diverted for non-forest purposes such as construction of dams, mining
and

other developmental activities only if the government permits.

• Since this diversion of forest land results in loss of biodiversity which in turn affects wildlife as well
as

geographical parameters such as climate and terrain, compensatory afforestation is also mandated
under

the Forest (Conservation) Act, 1980 that an equivalent area of non-forest land has to be taken up for

compensatory afforestation. Hence, option (b) is the correct answer.


• In addition to this, funds for raising the forest are also to be imposed on whomsoever is
undertaking the

diversion. The land chosen for afforestation, if viable, must be in close proximity of reserved or
protected

forest for ease of management by forest department.

• Compensatory Afforestation Fund Act seeks to provide an appropriate institutional mechanism,


both at

the Centre and in each State and Union Territory, to ensure expeditious utilization in efficient and

transparent manner of amounts released in lieu of forest land diverted for non-forest purpose which
would

mitigate impact of diversion of such forest land.

40. Consider the following statements regarding

the Swachh Sarvekshan 2021 Report:

1. The survey is conducted by Ministry of

Housing and Urban Affairs.

2. The survey is conducted annually both

in rural and urban areas.

Which of the statements given above is/are

correct?

(a) 1 only

(b) 2 only

(c) Both 1 and 2

(d) Neither 1 nor 2

Q 40.A

• Recently, Swachh Survekshan 2021 report has been released by Ministry of Housing and Urban

Affairs (MoHUA). Hence, statement 1 is correct.

• It is an annual survey of cleanliness, hygiene and sanitation in cities and towns across India under
Swachh
Bharat Mission-Urban (SBM-U).

• The first survey, which was undertaken in 2016, covered 73 cities; by 2021, in its sixth edition, the
survey

was grown to cover 4320 cities and was said to be the largest cleanliness survey in the world.

• MoHUA conducted the sixth edition of the survey to rank all cities under Swachh Bharat Mission-
Urban

(SBM-U) with Quality Council of India (QCI) as its implementation partner in a bid to scale up the

coverage of the ranking exercise and encourage towns and cities to actively implement mission
initiatives

in a timely and innovative manner.

• Swachh Survekshan is a pan India annual cleanliness survey that ranks India’s cities, towns and

states based on sanitation, waste management and overall cleanliness. Hence, statement 2 is not
correct.

41. Which of the following best describes the

technology called Selective Catalytic

Reduction (SCR)?

(a) It is a technology that uses ammonia to

convert NOx to nitrogen.

(b) It is a technology that uses oxygen in

excess to convert the harmful carbon

monoxide to carbon dioxide.

(c) It is a method that uses electric charge to

remove impurities in air.

(d) It is a technology that reduces sulphur

dioxide to sulphur.

Q 41.A

• Selective catalytic reduction (SCR)-It is currently the most widely applied technology in which
Ammonia
is used as a reducing agent to convert NOx to nitrogen in the presence of a catalyst in a converter.

• The catalyst is usually a mixture of titanium dioxide, vanadium pentoxide, and tungsten trioxide.

Hence option (a) is the correct answer. SCR can remove 60–90% of NOx from flue gases.

• The process is very expensive and the associated ammonia injection results in an ammonia
slipstream in

the exhaust.

42. With reference to Black Carbon, consider

the following statements:

1. It is a gaseous substance that results

from the incomplete combustion of

fossil fuels and biomass.

2. It remains in the atmosphere for a

shorter period of time and contributes to

climate change.

3. It lowers the albedo of Earth‟s surface

that causes temperature rise.

Which of the statements given above is/are

correct?

(a) 2 only

(b) 1 and 3 only

(c) 2 and 3 only

(d) 1, 2 and 3

Q 42.C

• Black carbon is not a gas but a particulate — a component of soot emitted by the incomplete

combustion of fossil fuels and biomass. Hence statement 1 is not correct.

• Along with methane, black carbon is one of the two greenhouse contributors the global
community has
targeted for immediate reduction to curb global warming before the Arctic melts.

• It stays in the atmosphere for shorter period of time and descends along with precipitation after
some

time. Because of black carbon’s short atmospheric life span, decreasing its presence offers an

opportunity to mitigate the effects of global warming quickly — within weeks. Hence statement 2 is

correct.

• Control of black carbon, according to many scientists — particularly from fossil-fuel sources —
could be

the fastest method of slowing global warming in the near future • Here on Earth, the albedo effect
has a significant impact on our climate. correct. The albedo of a surface is

the fraction of the incident sunlight that the surface reflects. The lower the albedo, the more
radiation

from the Sun that gets absorbed by the planet, and temperatures will rise. If the albedo is higher,
and

the Earth is more reflective, more of the radiation is returned to space, and the planet cools.

• Black carbon is generally thought to have both a direct warming effect (by absorbing incoming
solar

radiation in the atmosphere and converting it to heat radiation) and an indirect warming effect (by

reducing the reflectivity (albedo) of snow and ice). Hence statement 3 is correct.

43. Consider the following statements regarding

Sea Snot :

1. It refers to waste material released by

whales during mass migration.

2. It is highly beneficial to marine

ecosystem.

Which of the statements given above is/are

correct?

(a) 1 only

(b) 2 only
(c) Both 1 and 2

(d) Neither 1 nor 2

Q 43.D

• Recent context: Over the past few months, Turkey's Marmara Sea has been facing a new challenge.
The

Marmara sea that connects the Black Sea to the Aegean Sea has been blanketed with 'sea snot'
causing

new environmental concerns.

• Sea Snot was first discovered in Turkey in 2007. Also known as ‘marine mucilage’, it is a thick, slimy

grey-brown sheet that is formed by dead and living organic material. The sludge forms when algae

overloaded with nutrients feast on warm weather. This warm weather is caused due to global

warming. Hence, statement 1 is not correct.

• Environmental experts have said that the overproduction of phytoplankton caused by climate

change and the uncontrolled dumping of household and industrial waste into the seas has led to the

present crisis.

• The growth of the Sea Snot, which floats upon the surface of the sea like brown phlegm, is posing a

severe threat to the marine ecosystem of the country. It has caused mass deaths among the fish
population,

and also killed other aquatic organisms such as corals and sponges. Hence, statement 2 is not
correct.

• Ecologists said that the brown mucilage floating in the seas of Turkey is a sign of how the marine

ecosystem can be damaged and the effect it can have on the environment as a whole if serious steps
are

not taken to tackle the twin crisis of pollution and global warming.

44. Consider the following statements with

reference to Chemical Oxygen Demand

(COD):

1. It represents the amount of oxygen

required by microorganisms to break


down the organic matter present in water

at a specified temperature.

2. COD measurements are used as an

indicator of the size of a wastewater

treatment plant required for a specific

location.

Which of the statements given above is/are

correct?

(a) 1 only

(b) 2 only

(c) Both 1 and 2

(d) Neither 1 nor 2

Q 44.B

• Chemical Oxygen Demand (COD) is a test that measures the amount of oxygen required to

chemically oxidize the organic material and inorganic nutrients, such as Ammonia or Nitrate,

present in water. Hence statement 1 is not correct.

• The most commonly used oxidant is potassium dichromate, which is used in combination with
boiling

sulphuric acid. It is important to note that the chemical oxidant is not specific to organic or inorganic

compounds, hence both these sources of oxygen demand are measured in a COD assay.

• COD is an important water quality parameter and is used in a wide range of applications,
including:-to

confirming wastewater discharge and the waste treatment procedure meets criteria set by
regulators;-to

quantify the biodegradable fraction of wastewater effluent - the ratio between BOD and COD;-COD
or

BOD measurements are also used as an indicator of the size of a wastewater treatment plant

required for a specific location. Hence statement 2 is correct.


• Challenges associated with COD monitoringo Despite the test being entrenched in legislation there
are numerous problems and challenges

associated with the use of the test:-There is a lag until results are available (transportation to lab +
2h

for the test), hence environmental damage can occur before the data is available;-the test is
timeconsuming and expensive;-the test involves dangerous chemicals that need careful disposal and
are

potentially harmful to operators;-it fails to recreate natural processes (i.e. the test involves an
artificial

incubation with a strong oxidizing agent);-it is imprecise and has a high minimum detection limit thus

is not applicable to clean/uncontaminated river samples;

o It is clear that a move from traditional laboratory testing to in-situ (real-time) monitoring would

help to alleviate some of the problems outlined above. This is because COD is an ex situ method

since the sample is tested in labs.

o It would immediately address points I - III and would help to improve the spatial-temporal
resolution

of monitoring that would be directly beneficial to basin managers, water companies, and legislators

alike

45. Consider the following statements regarding

Arsenic pollution:

1. Arsenic has no smell or taste.

2. Arsenic is a carcinogenic agent.

3. Arsenic is the most common heavy

metal pollutant found in India's rivers.

Which of the statements given above is/are

correct?

(a) 1 only

(b) 1 and 2 only

(c) 2 and 3 only


(d) 1, 2 and 3

Q 45.B

• Arsenic is a natural component of the earth’s crust and is widely distributed throughout the
environment

in the air, water and land. It is highly toxic in its inorganic form.

• People are exposed to elevated levels of inorganic arsenic through drinking contaminated water,
using

contaminated water in food preparation and irrigation of food crops, industrial processes, eating

contaminated food and smoking tobacco.

• Long-term exposure to inorganic arsenic, mainly through drinking water and food, can lead to
chronic

arsenic poisoning. Skin lesions and skin cancer are the most characteristic effects.

• Arsenic doesn’t have a taste or odor and one can be exposed to it without knowing.

• Hence, statement 1 is correct.

• Health effects

o The International Agency for Research on Cancer (IARC) has classified arsenic and arsenic

compounds as carcinogenic to humans.

• Hence, statement 2 is correct

o The immediate symptoms of acute arsenic poisoning include vomiting, abdominal pain and
diarrhoea.

These are followed by numbness and tingling of the extremities, muscle cramping and death, in

extreme cases.

o The first symptoms of long-term exposure to high levels of inorganic arsenic (for example, through

drinking water and food) are usually observed in the skin and include pigmentation changes, skin

lesions and hard patches on the palms and soles of the feet (hyperkeratosis). These occur after a

minimum exposure of approximately five years.

• As per Central Water Commission (CWC), the most common heavy metal pollutant found in

India's rivers is iron.

• Hence, statement 3 is not correct.


46. In the context of the environment, „Carbon

Pricing‟ is best described as

(a) increase in the emission outside a region

as a direct result of the policy measures

to cap emissions in the region.

(b) the practice of estimating the

environmental cost of organic entities in

terms of the amount of carbon present in

them.

(c) tax evasion mechanisms used by

emitters to avoid paying taxes for

greenhouse gas emissions.

(d) shifting of burden for damage from

GHG emissions by assigning the

external costs to the emissions emitted

by any entity.

Q 46.D

• Carbon Pricing is an instrument that captures the external costs of greenhouse gas (GHG)
emissions -

the costs of emissions that the public pays for, such as damage to crops, health care costs from
heatwaves

and droughts, and loss of property from flooding and sea-level rise - and ties them to their sources
through

a price, usually in the form of a price on the carbon dioxide (CO2) emitted.

• A price on carbon helps shift the burden for the damage from GHG emissions back to those who
are

responsible for it and who can avoid it. Instead of dictating who should reduce emissions where and

how, a carbon price provides an economic signal to emitters, and allows them to decide to either
transform their activities and lower their emissions or continue emitting and paying for their

emissions. Hence option (d) is the correct answer.

• Other concepts:

o Carbon leakage refers to the situation that may occur if, for reasons of costs related to climate

policies, businesses were to transfer production to other countries with laxer emission constraints.

This could lead to an increase in their total emissions

o Carbon Tax - A carbon tax is a fee imposed on businesses and individuals that works as a sort of

"pollution tax." The tax is a fee imposed on companies that burn carbon-based fuels, including coal,

oil, gasoline, and natural gas.

o Carbon Trading is the buying and selling of credits that permit a company or other entity to

emit a certain amount of carbon dioxide. Carbon trade agreements allow for the sale of credits to

emit carbon dioxide between nations as part of an international agreement aimed at gradually

reducing total emissions.

o Carbon trading began under the Kyoto Protocol of 1997 (which came into force in 2005). Under

this, ‘certified emission reductions’ or CERs, were issued to entities that put up projects that reduced

emissions — such as wind, solar, or energy efficiency.

o It allows nations which are unable to meet their reduction targets to purchase carbon credits. Paris

Agreement also allows voluntary trading between countries to meet their NDC goals. If a country

reduces more GHG emissions than its target, it can sell the emission reduction to another country as

an “internationally traded mitigation outcome”.

o Zero Carbon Law by New Zealand - The Zero Carbon Act puts in place targets to reduce all

greenhouse gases: Carbon dioxide and nitrous oxide have to reduce to net zero by 2050.

47. An innovative Bubble Curtain Technology

was deployed in the Yamuna river to:

(a) monitor the migration of Ganges river

dolphin
(b) stop plastic from entering the Yamuna

river

(c) prevent seasonal foaming of Yamuna

river

(d) beautify the heritage places along the

Yamuna river stretch

Q 47.B

• Geocycle, the global waste management arm of building solutions provider LafargeHolcim, is

implementing innovative bubble curtain technology for the first time in India to stop plastic from
entering

the river Yamuna. The bubble barrier has been established on the Mantola canal of Agra city which

carries 40 percent of its storm and wastewater.

• A bubble curtain is a non-invasive solution to stop plastic from entering the oceans. Ships and fish

can pass through the air bubbles, but plastics will be stopped. The bubble screen is created by a

specially designed air tube which is placed diagonally on the bed of the canal or river. It brings

waste to the surface, channels the plastic onto the banks where it can be extracted. Hence, option
(b)

is the correct answer.

• The bubble barrier is generated using compressed air passing through tubes that are placed at the
bottom

of the canal. These tubes are connected with a compressor which is powered by renewable solar
energy.

Further, aeration in the canal will increase dissolved oxygen levels in the wastewater/stormwater
leading

to an overall improvement in the water quality.

48. With reference to the Blue Carbon

Ecosystem, consider the following

statements:
1. The carbon sequestering capacity of a

coastal blue carbon ecosystem is more

than those of tropical forests.

2. The Blue Carbon Initiative for securing

the Blue Carbon Ecosystem is an

initiative of Conservation International.

Which of the statements given above is/are

correct?

(a) 1 only

(b) 2 only

(c) Both 1 and 2

(d) Neither 1 nor 2

Q 48.C

• The coastal ecosystems of mangroves, tidal marshes, and seagrass meadows provide numerous
benefits

and services that are essential for climate change adaptation along coasts globally, including
protection

from storms and sea-level rise, prevention of shoreline erosion, regulation of coastal water quality,

provision of habitat for commercially important fisheries and endangered marine species, and food

security for many coastal communities. Additionally, these ecosystems sequester and store
significant

amounts of coastal blue carbon from the atmosphere and ocean and hence are now recognized for

their role in mitigating climate change.

• Coastal blue carbon ecosystems are found along the coasts of every continent except

Antarctica. Mangroves grow in the intertidal zone of tropical and subtropical shores. Countries with
the

highest areas of mangroves include Indonesia, Australia, Mexico, Brazil, Nigeria, Malaysia, Myanmar,

Papua New Guinea, Cuba, India, Bangladesh, and Mozambique.


• The Coastal Blue Carbon Ecosystems are very efficient at sequestering and storing carbon- each
square

mile of these systems can remove carbon from the atmosphere and oceans at a rate higher than
each

square mile of a tropical forest. Hence statement 1 is correct

• The Blue Carbon Initiative is the first integrated program focused on mitigating climate change by

conserving, restoring, and promoting sustainable use of coastal marine ecosystems globally. The
Initiative

currently focuses on mangroves, tidal marshes, and seagrasses. It is an international collaborative


effort

of the Conservation International, International Union for Conservation of Nature, and the

Intergovernmental Oceanic Commission of the UNESCO. Hence statement 2 is correct

49. Recently the 26th Conference of the Parties

(COP 26) to the UNFCCC was held. In this

context, consider the following statements

with reference to COP:

1. Conference of Parties is the supreme

decision-making body of the United

Nations Framework Convention on

Climate Change.

2. Presidency of COP rotates among the

five permanent members of the UN

Security Council.

Which of the statements given above is/are

correct?

(a) 1 only

(b) 2 only

(c) Both 1 and 2


(d) Neither 1 nor 2

Q 49.A

• Current Context: The 2021 United Nations Climate Change Conference, more commonly referred
to as

COP26, was the 26th United Nations Climate Change Conference (UNFCCC). It was held at the SEC

Centre in Glasgow, Scotland, United Kingdom.

• The conference was the 26th Conference of the Parties (COP) to the United Nations Framework

Convention on Climate Change

• Conference of the Parties (COP)

o The COP is the supreme decision-making body of the Convention. Hence, statement 1 is correct.

o All States that are Parties to the Convention are represented at the COP, at which they review the

implementation of the Convention and any other legal instruments that the COP adopts and take

decisions necessary to promote the effective implementation of the Convention, including


institutional

and administrative arrangements.

o A key task for the COP is to review the national communications and emission inventories
submitted

by Parties. Based on this information, the COP assesses the effects of the measures taken by Parties

and the progress made in achieving the ultimate objective of the Convention.

o The COP meets every year, unless the Parties decide otherwise. The COP meets in Bonn, the seat
of

the secretariat, unless a Party offers to host the session. The first COP meeting was held in Berlin,

Germany in 1995.

o The COP Presidency rotates among the five recognized UN regions - that is, Africa, Asia, Latin

America and the Caribbean, Central and Eastern Europe and Western Europe and Others. Also, there

is a tendency for the venue of the COP to also shift among these groups. Hence, statement 2 is not

correct.

50. Consider the following statements with


regard to installed renewable energy

capacity in India:

1. India's rank in renewable power installed

capacity is at 4th place globally.

2. India is top-ranked in installed solar

power capacity.

3. The government plans to establish a

renewable energy capacity of 1000 GW

by 2030.

Which of the statements given above is/are

correct?

(a) 1 only

(b) 2 and 3 only

(c) 1, 2 and 3

(d) 2 only

Q 50.A

• India's installed renewable power generation capacity has gained pace over the past few years
growing at

the rate of 17.33% between 2016 and 2020. The sector has become attractive from investors'
perspectives

as the government is increasing its support. The government is aiming to achieve 227 GW of
renewable

energy capacity (including 114 GW of solar capacity addition and 67 GW of wind power capacity) by

2022, more than its 175 GW target as per the Paris Agreement. The government plans to establish a

renewable energy capacity of 523 GW (including 73 GW from Hydro) by 2030. Hence statement 3 is
not

correct.

• The Indian renewable energy sector is the fourth most attractive renewable energy market in the
world.
India was ranked fourth in wind power, fifth in solar power, and fourth in renewable power installed

capacity, as of 2020. Hence statement 1 is correct and statement 2 is not correct.

51. Consider the following statements regarding

ozone pollution in India:

1. Ground-level ozone is formed by the

chemical reactions between oxides of

nitrogen (NOx) and volatile organic

compounds (VOC).

2. Ozone can reduce the yields of cereal

crops like wheat.

3. Ozone pollution is measured through

National Air Quality Index.

Which of the statements given above is/are

correct?

(a) 1 only

(b) 1 and 3 only

(c) 2 and 3 only

(d) 1, 2 and 3

Q 51.D

• Ozone is a gas composed of three atoms of oxygen. Ozone occurs both in the Earth's upper
atmosphere

and at ground level. Ozone can be good or bad, depending on where it is found.

• Ozone at ground level is a harmful air pollutant, because of its effects on people and the
environment, and

it is the main ingredient in “smog."

• Tropospheric, or ground-level ozone, is not emitted directly into the air but is created by chemical

reactions between oxides of nitrogen (NOx) and volatile organic compounds (VOC).
• This happens when pollutants emitted by cars, power plants, industrial boilers, refineries, chemical
plants,

and other sources chemically react in the presence of sunlight.

• Surface ozone is destroying around 22 million tonnes (21%) of India’s wheat yield and 6.5 million

tonnes (6%) rice crop every year, a multi-institute study led by the Indian Institute of
TechnologyMadras (IIT-M) has revealed, with Punjab and Haryana alone accounting for losses of
16% and

11% for wheat and rice respectively.

o Like any other gas, surface ozone enters the plant leaves through its stomata as part of normal

atmospheric gas exchange. Upon uptake, it dissolves in the water present in the plant and further

reacts with other chemicals affecting photosynthesis and thereby crop yields.

• Health Effects of Ozone Pollution

o Cause coughing and sore or scratchy throat.

o Make it more difficult to breathe deeply and vigorously and cause pain when taking a deep breath.

o Inflame and damage the airways.

o Make the lungs more susceptible to infection.

o Aggravate lung diseases such as asthma, emphysema, and chronic bronchitis.

o Increase the frequency of asthma attacks.

• National Air Quality Index (AQI) was launched by the Environment Ministry in April 2015.

• There are six AQI categories, namely Good, Satisfactory, Moderately polluted, Poor, Very Poor, and

Severe.

• The AQI considers eight pollutants (PM10, PM2.5, NO2, SO2, CO, O3, NH3, and Pb) for which

short-term (up to 24-hourly averaging period) National Ambient Air Quality Standards are

prescribed.

• Hence option (d) is the correct answer.

52. Which of the following is/are the

instruments related to carbon credit markets?

1. Certified Emission Reduction (CER)


2. Emission Reduction Unit (ERU)

3. Verified Emission Reduction (VER)

Select the correct answer using the code

given below.

(a) 1 and 2 only

(b) 2 and 3 only

(c) 3 only

(d) 1, 2 and 3

Q 52.D

There are two types of carbon markets:

• Regulatory compliance markets

o The compliance market is used by companies and governments that by law have to account for
their

GHG emissions. It is regulated by mandatory national, regional or international carbon reduction

regimes.

• Voluntary markets

o In the voluntary market, the trade of carbon credits is on a voluntary basis.

o Voluntary carbon credits (VER) are mainly purchased by the private sector.

o Corporate social responsibility (CSR) and public relations are the most common motivations for

buying carbon credits. Other reasons are considerations such as certification, reputation and

environmental and social benefits.

o Carbon credits on the voluntary markets are called Verified Emission Reductions (VER). Hence

option 3 is correct.

• Under Kyoto Protocol, two major mechanisms of regulated carbon trading markets are:

o Clean Development Mechanism (CDM

▪ Under the Kyoto Protocol developing countries (Non Annex I countries) are not obliged to reduce

their GHG emissions, whereas industrialized countries (Annex I countries) have to fulfill

specified targets.
▪ They can achieve these by reducing GHG emissions in their own country; implementing projects

to reduce emissions in other countries; or trading.

▪ An industrialized country implements an emission reduction project in a developing country. This

can be afforestation, energy efficiency or a renewable energy project. Because of the uptake or

savings of GHGs, carbon credits (CER) are generated.

▪ The carbon credits from CDM projects are called Certified Emission Reductions (CER). Hence,

option 1 is correct.

o Joint Implementation (JI)

▪ Any Annex I country can invest in a project to reduce greenhouse gas emissions in any other

Annex I country (referred to as a "Joint Implementation Project") as an alternative to reducing

emissions domestically.

▪ In this way countries can lower the costs of complying with their Kyoto targets by investing in

projects where reducing emissions may be cheaper and applying the resulting Emission

Reduction Units (ERUs) towards their commitment goal. Hence, option 2 is correct.

▪ The emission reduction unit (ERU) is an emissions unit issued under a Joint Implementation

project.

53. Which of the following methods can be

potentially adopted as a part of geoEngineering techniques to avoid Global

Warming?

1. Shooting of mirrors into space

2. Firing silver iodide into clouds to

produce rain

3. Seeding the sea with iron particles

4. Injection of sulphur particles in the sky

Select the correct answer from the code

given below.

(a) 2, 3 and 4 only


(b) 1 and 3 only

(c) 1, 2, 3 and 4

(d) 2 and 4 only

Q 53.C

• Geoengineering is the intentional manipulation of the climate system to counteract global climate
change.

Scientists have speculated on a wide array of geoengineering schemes that fall into two sets of
strategies.

One set aims at massive efforts to remove CO2 from the atmosphere to reduce the greenhouse

effect. An example of this type of strategy is to engineer machines, "artificial trees," that capture

carbon from the ambient air and sequester it, for instance, in abandoned oil or gas fields. The other
set

of strategies has been called solar radiation management (SRM). This general approach aims to cool

the earth by reflecting incoming solar radiation, either in the upper atmosphere or at or near the
earth's

surface.

Some of the proposed methods of Geo-Engineering to avoid Global Warming and mitigate climate
change are

as follows:

• Blasting sulfate particles into the upper atmosphere to reflect the sun’s rays;

• Dumping iron particles in the oceans to nurture CO2-absorbing plankton;

• Firing silver iodide into clouds to produce rain;

• Shooting mirrors into space to deflect enough sunlight to bring Earth's climate back to its pre-
industrial

level

• Genetically engineering crops to have reflective leaves;

• Spraying seawater into clouds to make clouds whiter;

• Dumping large quantities of plant matter into the ocean or turning it into charcoal for burying in
soils.

• Hence option (c) is the correct answer


54. With reference to Rotterdam Convention,

consider the following statements:

1. It is a legally binding multilateral treaty

that seeks to promote shared

responsibilities in relation to the

importation of hazardous chemicals.

2. In India, the Department of Chemicals

and Petrochemicals is the Designated

National Authority for pesticides.

Which of the statements given above is/are

correct?

(a) 1 only

(b) 2 only

(c) Both 1 and 2

(d) Neither 1 nor 2

Q 54.A

• The Rotterdam Convention (formally, the Rotterdam Convention on the Prior Informed Consent

Procedure for Certain Hazardous Chemicals and Pesticides in International Trade) is a legally binding

multilateral treaty. The Convention seeks to promote shared responsibility and cooperative efforts

among State Parties in the international trade of certain hazardous chemicals in order to protect

human health and the environment from potential harm. Hence, statement 1 is correct.

• The convention promotes open exchange of information and calls on exporters of hazardous
chemicals to

use proper labeling, include directions on safe handling, and inform purchasers of any known
restrictions

or bans.

• The Convention covers both pesticides and industrial chemicals that have been banned or severely

restricted for health or environmental reasons by Parties and which have been notified by Parties for
inclusion in the PIC procedure.

• Each Party is required to designate a National Authority for performing the administrative
functions

required under the Convention. In India, Department of Chemicals and Petrochemicals is the
Designated

National Authority (DNA) for industrial chemicals and Department of Agriculture and Co-operation is

the Designated National Authority (DNA) for pesticides. Hence, statement 2 is not correct.

55. Consider the following pairs:

Convention/

Agreement

Deals with

1. Basel Convention : Control of Transboundary

movements of Hazardous

waste

2. Stockholm Convention : Persistent Organic

Pollutants

3. Minamata convention : Anthropogenic release of

mercury

Which of the pairs given above is/are

correctly matched?

(a) 1 and 2 only

(b) 1, 2 and 3

(c) 3 only

(d) 2 and 3 only

Q 55.B

• The Minamata Convention on Mercury is a global treaty to protect human health and the

environment from the adverse effects of mercury. The Minamata Convention draws attention to a
global and ubiquitous metal that, while naturally occurring, has broad uses in everyday objects and is

released to the atmosphere, soil, and water from a variety of sources. Controlling the anthropogenic

releases of mercury throughout its lifecycle has been a key factor in shaping the obligations under
the

Convention. Hence pair 3 is correctly matched

• The Stockholm Convention on Persistent Organic Pollutants was adopted by the Conference of

Plenipotentiaries on 22 May 2001 in Stockholm, Sweden. The Convention entered into force on 17
May

2004. The Stockholm Convention on Persistent Organic Pollutants is a global treaty to protect human

health and the environment from chemicals that remain intact in the environment for long periods,
become

widely distributed geographically, accumulate in the fatty tissue of humans and wildlife, and have
harmful

impacts on human health or the environment. Hence pair 2 is correctly matched

• The Basel Convention on the Control of Transboundary Movements of Hazardous Wastes and their

Disposal was adopted on 22 March 1989 by the Conference of Plenipotentiaries in Basel,


Switzerland, in

response to a public outcry following the discovery, in the 1980s, in Africa and other parts of the

developing world of deposits of toxic wastes imported from abroad. The overarching objective of the

Basel Convention is to protect human health and the environment against the adverse effects of
hazardous

wastes. Its scope of application covers a wide range of wastes defined as “hazardous wastes” based
on

their origin and/or composition and their characteristics, as well as two types of wastes defined as
“other

wastes” - household waste and incinerator ash. Hence pair 1 is correctly matched

56. Consider the following statements regarding

the Coastal Regulation Zones (CRZs):

1. CRZs are declared by the Ministry of

Environment, Forest and Climate


Change.

2. The CRZs rules include only the intertidal zone and land part of the coastal

area.

Which of the statements given above is/are

correct?

(a) 1 only

(b) 2 only

(c) Both 1 and 2

(d) Neither 1 nor 2

Q 56.A

• Recently, a draft notification has been issued proposing amendments to the Coastal Regulation

Zone (CRZ) notifications, 2019.

• Key changes proposed:

o Development and production of oil and natural gas and shall be exempted from prior CRZ
clearance.

o Purely temporary and seasonal structures (shacks) may be retained during the monsoon season
with

adequate precautions.

o Till Integrated Island Management Plan (IIMP) are formulated by respective states/UTs, CRZ

notification 2011 shall continue to apply

o The sand bars in the intertidal areas shall be removed by traditional coastal communities only

by manual method.

These are proposed changes not yet made official.

• Under the Environment Protection Act, 1986 a notification was issued in February, 1991, for

regulation of activities in the coastal area by the Ministry of Environment and Forests (MoEF).

Hence, statement 1 is correct.

• Coastal Regulation Zones:

o CRZ-1 allows eco-tourism activities such as mangrove walks, tree huts, nature trails, in eco-
sensitive
areas, demarcated as CRZ-IA. Sea links, salt harvesting and desalination plants and roads on stilts are

also allowed in CRZ-IA. The controversial land reclamation, in which new land is created from

oceans or lake beds and is known to have strong impacts on coastal ecology, has been allowed in

intertidal or CRZ-IB areas, for ports and sea links.

o CRZ-II includes a substantially built-up area, project developers can now increase the floor area
ratio

or floor space index, and build resorts and other tourism facilities.

o CRZ 3 areas includes land areas that are relatively undisturbed and those which do not fall under
CRZ

2, are categorized into CRZ 3A and 3B based on the density of population.


o CRZ-IV, includes the shallow belt of coastal waters extending up to 12 nautical miles. Hence,

statement 2 is not correct.

57. Which of the following correctly describes

'Carbon Intensity Indicator'?

(a) Amount of carbon emissions by a power

plant per unit of energy produced.

(b) Amount of carbon emissions generated

by a healthy human being during his/her

lifetime.

(c) Amount of carbon emissions generated

by a Ship in doing one unit of transport

work.

(d) Potential loss of carbon sequestration

due to rapid melting of glacial sheets.

Q 57.C

• The Carbon Intensity Indicator (CII) is a measure of how efficiently a ship transports goods or

passengers and is given in grams of CO2 emitted per cargo-carrying capacity and nautical mile.

Hence, option (c) is the correct answer.


• The ship is then given an annual rating ranging from A to E, whereby the rating thresholds will
become

increasingly stringent towards 2030.

• The CII applies to all cargo and cruise ships above 5,000 GT (Giga Tonne).

• The CII is based directly on fuel consumption, which is influenced by how a specific ship is operated
in

combination with its technical efficiency and fuel.

• Its value will be affected by the type of fuel used, the efficiency of the vessel and operational
parameters

such as vessel speed, cargo transported, weather conditions and the general condition of the vessel
(e.g.

biofouling).

• The yearly CII is calculated based on reported International Maritime Organisation (IMO) data.

58. Consider the following statements about

Methanol:

1. It has a much higher energy efficiency

than petrol and diesel.

2. It burns efficiently in all internal

combustion engines.

3. It does not produce any particulate

matter on burning.

Which of the statements given above is/are

correct?

(a) 1 only

(b) 1 and 3 only

(c) 2 and 3 only

(d) 1, 2 and 3

Q 58.C
• Methanol is a low carbon, hydrogen carrier fuel produced from high ash coal, agricultural residue,
CO2

from thermal power plants, and natural gas. It is the best pathway for meeting India’s commitment
to COP

21. NITI Aaayog has drawn out a road map to substitute 10% of Crude imports by 2030, with
Methanol

alone.

• Statement 1 is not correct: Methanol has slightly lower energy efficiency (22.7 MJ/Kg) than petrol

(44-46 MJ/Kg) and diesel (42-46 MJ/Kg) however it can replace both these fuels in the transport
sector

(road, rail, and marine), the energy sector (comprising tractors and commercial vehicles), and retail

cooking (replacing kerosene and wood charcoal).

• Statement 2 is correct: Methanol burns efficiently in all internal combustion engines. The gaseous

version of Methanol – DME (Dimethyl Ether) can be blended with LPG and can be an excellent
substitute

for diesel in Large buses and trucks.

• Statement 3 is correct: Methanol produces no particulate matter, no soot, almost nil SOX and NOX

emissions (Near Zero Pollution). It can be produced from Natural Gas, Indian High Ash Coal, Bio-
mass,

MSW, stranded and flared gases, etc.

59. As per National policy on biofuels, which of

the following raw materials can be used for

ethanol production?

1. Sugar Beet

2. Sweet Sorghum

3. Cassava

4. Barley seeds

5. Broken rice

6. Chickpea
Select the correct answer using the code

given below.

(a) 1, 2, 3 and 5 only

(b) 2, 4, and 6 only

(c) 1, 3, 4 and 6 only

(d) 1, 2, 3, 4, 5 and 6

Q 59.A

• The National Biofuel Policy 2018 categorizes biofuels as "Basic Biofuels" viz. First Generation (1G)

bioethanol & biodiesel and "Advanced Biofuels" - Second Generation (2G) ethanol, Municipal Solid

Waste (MSW) to drop-in fuels, Third Generation (3G) biofuels, bio-CNG, etc. to enable the extension
of

appropriate financial and fiscal incentives under each category.

• The Policy expands the scope of raw material for ethanol production by allowing the use of

Sugarcane Juice, Sugar containing materials like Sugar Beet, Sweet Sorghum, Starch containing

materials like Corn, Cassava, Damaged food grains like wheat, broken rice, Rotten Potatoes, unfit

for human consumption for ethanol production.

• With a thrust on Advanced Biofuels, the Policy indicates a viability gap funding scheme for 2G
ethanol

Bio refineries of Rs.5000 crore in 6 years in addition to additional tax incentives, higher purchase
price as

compared to 1G biofuels.

• It encourages the setting up of supply chain mechanisms for biodiesel production from non-edible

oilseeds, Used Cooking Oil, short gestation crops.

60. Which among the following are the

components of Green Crackers?

1. Lithium

2. Barium

3. Aluminium
4. Thermite

Select the correct answer using the code

given below.

(a) 1 and 2 only

(b) 1, 2 and 3 only

(c) 3 and 4 only

(d) 1, 3 and 4 only

Q 60.C

• Green Crackers are environmentally friendly fireworks and can reduce the air pollution caused by

traditional firecrackers. These have been developed by the National Environmental and Engineering

Research Institute (NEERI), a Council of Scientific and Industrial Research (CSIR) lab.

• Green crackers don't contain banned chemicals such as lithium, arsenic, barium and lead. They are

called Safe Water Releaser (SWAS), Safe Thermite Cracker (STAR) and Safe Minimal Aluminium

(SAFAL) crackers. Hence option (c) is the correct answer.

• These green crackers have been given three different names:

• SWAS – Safe Water Releaser-They will release water vapour in the air which will suppress the dust

released.

o It will not comprise potassium nitrate and sulphur.

o A diluent will be released for gaseous emissions.

o The particulate dust released will reduce by approximately 30 percent.

• STAR – Safe Thermite Cracker-Does not comprise potassium nitrate and sulphur.

o Reduced particulate matter disposal.-Reduced sound intensity.

• SAFAL – Safe Minimal Aluminium

o Minimum usage of aluminum.

o Usage of magnesium instead of aluminum.

o Reduction in sound in comparison to traditional crackers


61. With reference to stubble burning, consider

the following statements:

1. The oxides of sulfur are released during

crop residue burning.

2. The heat from burning paddy straw kills

the bacterial and fungal populations

critical for fertile soil.

3. Burning crop residue is a crime both

under the Indian Penal Code (IPC) and

under the Air Pollution Control Act of

1981.

Which of the statements given above are

correct?

(a) 1 and 2 only

(b) 2 and 3 only

(c) 1 and 3 only

(d) 1, 2 and 3

Q 61.D

• A study estimates that crop residue burning released 149.24 million tonnes of carbon dioxide
(CO2),

over 9 million tonnes of carbon monoxide (CO), 0.25 million tonnes of oxides of sulphur (SOX), 1.28

million tonnes of particulate matter and 0.07 million tonnes of black carbon. Hence statement 1 is

correct.

• These directly contribute to environmental pollution, and are also responsible for the haze in Delhi
and

melting of Himalayan glaciers.

• The heat from burning paddy straw penetrates 1 centimetre into the soil, elevating the
temperature to

33.8 to 42.2 degree Celsius. This kills the bacterial and fungal populations critical for a fertile soil.
Hence statement 2 is correct.

• Burning of crop residue causes damage to other micro-organisms present in the upper layer of the
soil as

well as its organic quality. Due to the loss of ‘friendly’ pests, the wrath of ‘enemy’ pests has
increased and

as a result, crops are more prone to disease. The solubility capacity of the upper layers of soil have
also

been reduced.

• According to a report, one tonne stubble burning leads to a loss of 5.5 kilogram nitrogen, 2.3 kg

phosphorus, 25 kg potassium and more than 1 kg of sulfur — all soil nutrients, besides organic

carbon.

• Burning crop residue is a crime under Section 188 of the IPC and under the Air and Pollution

Control Act of 1981. However, government’s implementation lacks strength. Hence statement 3 is

correct.

62. Sustainable Alternative Towards Affordable

Transportation (SATAT) Initiative is related

to:

(a) Fuel Cells

(b) Compressed Biogas

(c) Biodiesel

(d) Hybrid and Electronic Vehicles

Q 62.B

• Union Minister of Petroleum and Natural Gas & Skill Development and Entrepreneurship launched
an

innovative initiative with PSU Oil Marketing Companies (OMCs) from potential entrepreneurs to set

up Compressed Bio-Gas (CBG) production plants and make available CBG in the market for use in

automotive fuels. It is an alternative, green transport fuel for efficient management of biomass and
organic
waste.

• Bio-gas is produced naturally through a process of anaerobic decomposition from waste sources
like

agriculture residue, cattle dung, sugarcane press mud, municipal solid waste, sewage treatment
plant

waste, etc. After purification, it is compressed and called CBG, which has pure methane content of

over 95%. Compressed Bio-Gas is exactly similar to the commercially available natural gas in its

composition and energy potential. With calorific value (~52,000 KJ/kg) and other properties similar
to

CNG, Compressed Bio-Gas can be used as an alternative, renewable automotive fuel.

63. With reference to the Convention on

International Trade in Endangered Species

of Wild Fauna and Flora (CITES), consider

the following statements:

1. It is the only global treaty that ensures

the trade in plants and animals does not

threaten their survival in the wild.

2. CITES is legally binding on the parties

and it takes the place of national laws.

3. CITES is administered through the

United Nations Environment Programme

(UNEP).

Which of the statements given above is/are

correct?

(a) 1 only

(b) 2 and 3 only

(c) 1 and 3 only


(d) 1, 2 and 3

Q 63.C

• The Convention on International Trade in Endangered Species of Wild Fauna and Flora (CITES) is an

international agreement between governments that entered into force in 1975, and became the
only

treaty to ensure that international trade in plants and animals does not threaten their survival in

the wild. Hence, statement 1 is correct.

• CITES is an international agreement to which States and regional economic integration


organizations

adhere voluntarily. States that have agreed to be bound by the Convention ('joined' CITES) are
known as

Parties. Although CITES is legally binding on the Parties – in other words, they have to implement

the Convention – it does not take the place of national laws. Rather it provides a framework to be

respected by each Party, which has to adopt its own domestic legislation to ensure that CITES is

implemented at the national level. Hence, statement 2 is not correct.

• CITES is administered through the United Nations Environment Programme(UNEP). A Secretariat,

located in Geneva, Switzerland, oversees the implementation of the treaty and assists with

communications between countries. Hence, statement 3 is correct.

• Species for which trade is controlled are listed in one of three Appendices to CITES, each conferring
a

different level of regulation and requiring CITES permits or certificates.

64. Which of the following are the major effects

of ammonia pollution?

1. Eutrophication and Acidification effects

on semi-natural ecosystems

2. Dead zones with dangerously low

oxygen levels in the water bodies


3. Formation of haze like conditions in the

atmosphere

Select the correct answer using the code

given below.

(a) 1 and 2 only

(b) 2 and 3 only

(c) 1 and 3 only

(d) 1, 2 and 3

Q 64.D

• Ammonia (NH3) is a colorless highly reactive and soluble alkaline gas. It is a prominent constituent
of

the nitrogen cycle that adversely affects ecosystems at higher concentrations.

• Sources of emissions: o The largest source of NH3 emissions is agriculture, including animal
husbandry

and NH3-based fertilizer applications.

• Other sources of NH3 include industrial processes, vehicular emissions, volatilization from soils and

oceans, decomposition of organic waste, forest fires, animal and human waste, nitrogen fixation

processes.

• Effects of Ammonia Pollution:

o On Environment – Climate change – Nitrification and Denitrification contribute to greenhouse

gas emissions.

o Air Pollution – Formation of haze-like conditions due to reaction of Ammonia with other oxides
and

pollutants in the atmosphere. Hence option 3 is correct.

o Water Pollution – Toxic to aquatic organisms and contributes to harmful algal blooms and dead

zones with dangerously low oxygen levels. Hence option 2 is correct.

o Ecosystem damage – Eutrophication and acidification effects on semi-natural ecosystems and lead

to change in species composition. Hence option 1 is correct.

o In humans- Ammonia interacts with moisture present in the skin, eyes, oral cavity, respiratory tract
to form ammonium hydroxide, which is very caustic and disrupts the cell membrane, lipids,

ultimately leading to cellular destruction.

o Gaseous Ammonia reacts with other pollutants in the air to form fine particles of ammonia salts

which affect human breathing as well as cause diseases like pneumonia and asthma.

65. Consider the following statements with

reference to TRAFFIC, the Wildlife Trade

Monitoring Network:

1. It was established by International Union

for Conservation of Nature (IUCN) and

United Nations.

2. It is currently now an independent nonprofit organization.

Which of the statements given above is/are

correct?

(a) 1 only

(b) 2 only

(c) Both 1 and 2

(d) Neither 1 nor 2

Q 65.B

• TRAFFIC is an organization that was established in 1976 by World Wildlife Fund (WWF)

and International Union for Conservation of Nature (IUCN) as a wildlife trade monitoring network to

undertake data collection, analysis, and provision of recommendations to inform decision making on

wildlife trade. Hence statement 1 is not correct.

• For over 40 years TRAFFIC performed that function as a leader in wildlife trade research, as a joint

program of WWF and IUCN.

• TRAFFIC became an independent non-profit organization in 2017, with WWF and IUCN sitting on its

Board of Directors along with independent Board members. Hence statement 2 is correct.
• TRAFFIC is renowned globally for its expertise and influence in the wildlife trade and conservation

arena, as a provider of objective and reliable information. Its expert staff implement innovative
projects

and create new tools to deliver the mission of protecting nature and supporting sustainable
development,

by resolving wildlife trade challenges.

• TRAFFIC is an international network, consisting of TRAFFIC International, based in Cambridge, UK

with offices on five (Africa, Asia, Europe, North and South America) continents.

66. Consider the following statements regarding

the Convention on Conservation of

Migratory Species (CMS):

1. CMS is the only global

intergovernmental organisation

established exclusively for the

conservation and management of

migratory species.

2. It is an environmental treaty under the

aegis of the United Nations Environment

Programme.

Which of the statements given above is/are

correct?

(a) 1 only

(b) 2 only

(c) Both 1 and 2

(d) Neither 1 nor 2

Q 66.C

• The Convention on the Conservation of Migratory Species of Wild Animals (Bonn Convention; CMS)
is
an environmental treaty under the aegis of the United Nations Environment Programme. Hence,

statement 2 is correct.

• It provides a global platform for the conservation and sustainable use of migratory animals and
their

habitats. CMS is the only global and UN-based intergovernmental organisation established

exclusively for the conservation and management of terrestrial, aquatic and avian migratory species

throughout their range. Hence, statement 1 is correct.

• Migratory species threatened with extinction are listed on Appendix I of the Convention. CMS
Parties

strive towards strictly protecting these animals, conserving or restoring the places where they live,

mitigating obstacles to migration and controlling other factors that might endanger them. Besides

establishing obligations for each State joining the Convention, CMS promotes concerted action
among the

Range States of many of these species.

• Migratory species that need or would significantly benefit from international co-operation are
listed in

Appendix II of the Convention. For this reason, the Convention encourages the Range States to
concludes

global or regional agreements.

You might also like